Line 4: Line 4:
  
 
Are the boundary conditions for this problem u(0,t)=U1 and u(L,t)=U2 or u'(0,t)=U1 and u'(L,t)=U2?  I think it is the u' option, but I'm not sure why?
 
Are the boundary conditions for this problem u(0,t)=U1 and u(L,t)=U2 or u'(0,t)=U1 and u'(L,t)=U2?  I think it is the u' option, but I'm not sure why?
 +
 +
Answer: The problem says that it is fixed at U1 and U2 for all time. I interpret this as X(0)=U1 and X(L)=U2, and X'(0)=X'(L)=0 since they are not changing at those points. Apply these to the X equation after you separate your variables. The T equation will come out the same as before.
  
  

Revision as of 10:08, 4 December 2010

Homework 14 collaboration area

Question Page 560, Problem 11:

Are the boundary conditions for this problem u(0,t)=U1 and u(L,t)=U2 or u'(0,t)=U1 and u'(L,t)=U2? I think it is the u' option, but I'm not sure why?

Answer: The problem says that it is fixed at U1 and U2 for all time. I interpret this as X(0)=U1 and X(L)=U2, and X'(0)=X'(L)=0 since they are not changing at those points. Apply these to the X equation after you separate your variables. The T equation will come out the same as before.


Question Page 585, Prob 6:

Can anyone provide some direction on how to start this problem? I'm not really sure how to get started on it.


Question Page 568, Prob 2:

What are the limits of integration for A(p) and B(p) in this problem. I think it is -infinity to infinity, but I'm not sure if this is correct.


Back to the MA 527 start page

To Rhea Course List

Alumni Liaison

Questions/answers with a recent ECE grad

Ryne Rayburn